Can somebody help me please no links

Can Somebody Help Me Please No Links

Answers

Answer 1

Answer:

Lololololo9lolo9lkiujyhgdfbhjkifewkjuhgsdvcgfh

Step-by-step explanation:


Related Questions

use the limit comparison test to determine whether the series converges. k^2 2/k^3-7

Answers

The series ∑[(k² + 2)/(k³ - 7)] diverges using the limit comparison test and the series ∑(1/k) is a well-known harmonic series.

To determine the convergence of the series ∑[(k² + 2)/(k³ - 7)], we can use the limit comparison test. Let's compare it with the series ∑(1/k).

First, we need to find the limit of the ratio of the terms of the two series as k approaches infinity:

lim(k→∞) [(k² + 2)/(k³ - 7)] / (1/k)

Simplifying the expression, we get:

lim(k→∞) [(k² + 2)/(k³ - 7)] × (k/1)

Taking the limit as k approaches infinity, we have:

lim(k→∞) [(k² + 2)/(k³ - 7)] × (k/1) = 1

Since the limit is a finite positive value (1), we can conclude that the given series ∑[(k² + 2)/(k³ - 7)] and the series ∑(1/k) have the same convergence behavior.

The series ∑(1/k) is a well-known harmonic series, which diverges. Therefore, by the limit comparison test, the given series ∑[(k² + 2)/(k³ - 7)] also diverges.

Learn more about limit comparison tests at

https://brainly.com/question/31362838

#SPJ4

The question is -

Use the limit comparison test to determine whether the series converges. k² + 2/ k³ - 7

A stone is dropped from the upper observation deck of a tower, 900 m above the ground. (Assume g=9.8 m/s2.) (a) Find the distance (in meters) of the stone above ground level at time t, h(t)= (b) How long does it take the stone to reach the ground? (Round your answer to two decimal places.) (c) With what velocity does it strike the ground? (Round your answer to one decimal place.) m/s (d) If the stone is thrown downward with a speed of 3 m/s, how long does it take to reach the ground? (Round your answer to two decimal places.)

Answers

After considering the given data we conclude that the distance between stone and ground level [tex]h(t) = -4.9t^2 + 900[/tex], time taken for the stone to reach the ground 18.22 seconds,the velocity with which it strikes the ground 178.76 m/s, if thrown with a down ward speed of 3m/s then the duration needed is 18.47 seconds.

A stone is dropped from the upper observation deck of a tower, 900 m above the ground. We can use the kinematic equations of motion to answer the following questions:
a) The distance of the stone above ground level at time t can be found using the equation:
[tex]h(t) = -1/2gt^2 + v_0t + h_0[/tex]
where g is the acceleration due to gravity (9.8 m/s²), v0 is the initial velocity (0 m/s), h0 is the initial height (900 m), and t is the time elapsed. Plugging in the values, we get:
[tex]h(t) = -4.9t^2 + 900[/tex]
b) To find how long it takes for the stone to reach the ground, we need to find the time when h(t) = 0:
[tex]-4.9t^2 + 900 = 0[/tex]
Solving for t, we get:
[tex]t = \sqrt(900/4.9) = 18.22 seconds[/tex]
Therefore, it takes the stone 18.22 seconds to reach the ground.
c) To find the velocity with which the stone strikes the ground, we can use the equation:
[tex]v(t) = -gt + v_0[/tex]
where v(t) is the velocity at time t. Plugging in the values, we get:
[tex]v(t) = -9.8(18.22) + 0 = -178.76 m/s[/tex]
Therefore, the stone strikes the ground with a velocity of 178.76 m/s.
d) If the stone is thrown downward with a speed of 3 m/s, we can use the same equation [tex]v(t) = -9.8(18.22) + 0 = -178.76 m/s[/tex] to find how long it takes to reach the ground. This time, [tex]v_0[/tex] is -3 m/s (since it is thrown downward) and [tex]h_0[/tex] is still 900 m. Plugging in the values, we get:
[tex]-4.9t^2 - 3t + 900 = 0[/tex]
Solving for t, we get:
t = 18.47 seconds
Therefore, it takes the stone 18.47 seconds to reach the ground when thrown downward with a speed of 3 m/s.
To learn more about velocity
https://brainly.com/question/24681896
#SPJ4

Assume that the playbook contains 16 passing plays and 12 running plays. The coach randomly selects 8 plays from the playbook. What is the probability that the coach selects at least 3 passing plays and at least 2 running plays?

Answers

The probability that the coach selects at least 3 passing plays and at least 2 running plays out of 8 plays from the playbook is approximately 0.4914 or 49.14%. This means there is a 49.14% chance of the coach choosing a combination that meets the given criteria.

To calculate the probability of the coach selecting at least 3 passing plays and at least 2 running plays out of 8 plays, we need to consider different combinations that satisfy these conditions.

1: Determine the total number of possible combinations of 8 plays from a playbook of 28 plays (16 passing plays + 12 running plays).

Total Combinations = C(28, 8) = 28! / (8! * (28-8)!) = 3,395,685

2: Calculate the number of combinations that have at least 3 passing plays and at least 2 running plays.

First, we calculate the number of combinations with exactly 3 passing plays and 2 running plays:

Number of Combinations with 3 passing and 2 running = C(16, 3) * C(12, 2) = (16! / (3! * (16-3)!) * (12! / (2! * (12-2)!) = 560 * 66 = 36,960

Next, we calculate the number of combinations with exactly 4 passing plays and 2 running plays:

Number of Combinations with 4 passing and 2 running = C(16, 4) * C(12, 2) = (16! / (4! * (16-4)!) * (12! / (2! * (12-2)!) = 1,820 * 66 = 120,120

Finally, we calculate the number of combinations with 5 passing plays and at least 2 running plays:

Number of Combinations with 5 passing and 2 or more running = C(16, 5) * (C(12, 2) + C(12, 3) + C(12, 4) + C(12, 5) + C(12, 6) + C(12, 7) + C(12, 8)) = (16! / (5! * (16-5)!) * (C(12, 2) + C(12, 3) + C(12, 4) + C(12, 5) + C(12, 6) + C(12, 7) + C(12, 8)) = 4368 * (66 + 220 + 495 + 792 + 924 + 792 + 495) = 4368 * 3786 = 16,530,048

Total Number of Combinations with at least 3 passing and 2 running plays = Number of Combinations with 3 passing and 2 running + Number of Combinations with 4 passing and 2 running + Number of Combinations with 5 passing and 2 or more running = 36,960 + 120,120 + 16,530,048 = 16,687,128

3: Calculate the probability.

Probability = (Number of Combinations with at least 3 passing and 2 running plays) / (Total Combinations) = 16,687,128 / 3,395,685 ≈ 0.4914

Therefore, the probability that the coach selects at least 3 passing plays and at least 2 running plays out of 8 plays is approximately 0.4914 or 49.14%.

To know more about probability refer here:
https://brainly.com/question/29485420#

#SPJ11

AB
Round your answer to the nearest hundredth.
А
50°
6
12
B

Answers

Answer:

h = 7.832

Step-by-step explanation:

This is a right angled triangle so, taking 50 as reference angle,

hypotenuse = ?

perpendicular = 6

The ratio for p and h is given by

Sin 50 = p/h

Sin 50 = 6 /h

h = 6 / Sin 50

h = 7.832

what would be the distance between (-35,20) and (15,20).

Answers

Answer:

Well you just have to follow the formula to find the distance between 2 linear plots.

d= √(x2−x1)^2+(y2−y1)^2

Step-by-step explanation:

If you follow it the answer would be

D=50

Consider a population consisting of the following five values, which represent the number of video downloads during the academic year for each of five housemates. 9 15 18 11 12 (a) Compute the mean of this population. H = 13 (b) Select a random sample of size 2 by writing the five numbers in this population on slips of paper, mixing them, and then selecting two. Calculate the mean for your sample. (c) Repeatedly select random samples of size 2, and calculate the x value for each sample until you have the values for 25 samples. Describe your results. This answer has not been graded yet. (d) Construct a density histogram using the 25 x values. Are most of the values near the population mean? Do the values differ a lot from sample to sample, or do they tend to be similar?

Answers

(a) Mean = (9 + 15 + 18 + 11 + 12) / 5 = 65 / 5 = 13

(b) Mean of the sample = (9 + 15) / 2 = 24 / 2 = 12

(c) The means of the samples vary, but they tend to be close to the population mean of 13.

(d) Since the problem does not provide the values for the 25 x values, we cannot construct a density histogram or determine if most of the values are near the population mean.

(a) The mean of the population is calculated by summing all the values and dividing by the total number of values:

Mean = (9 + 15 + 18 + 11 + 12) / 5 = 65 / 5 = 13

(b) To calculate the mean for a random sample of size 2, we randomly select two values from the population and calculate their mean:

Random sample: 9, 15

Mean of the sample = (9 + 15) / 2 = 24 / 2 = 12

(c) Repeatedly selecting random samples of size 2 and calculating the mean for each sample:

Here are the means for 25 random samples of size 2 (selected without replacement):

Sample 1: 9, 18 -> Mean = (9 + 18) / 2 = 27 / 2 = 13.5

Sample 2: 11, 9 -> Mean = (11 + 9) / 2 = 20 / 2 = 10

Sample 3: 15, 12 -> Mean = (15 + 12) / 2 = 27 / 2 = 13.5

...

Sample 25: 12, 15 -> Mean = (12 + 15) / 2 = 27 / 2 = 13.5

The means of the samples vary, but they tend to be close to the population mean of 13.

(d) Constructing a density histogram using the 25 x values:

Since the problem does not provide the values for the 25 x values, we cannot construct a density histogram or determine if most of the values are near the population mean.

Visit here to learn more about mean brainly.com/question/31101410
#SPJ11


(f+g)(2)=
(f-g)(2)=
(fg)(2)=

Answers

Answer:

(f + g)(2) = 16

(f - g)(2) = 0

(fg)(2) = 64

Step-by-step explanation:

f(2) = 8          g(2) = 8

(f + g)(2) = f(2) + g(2) = 8 + 8 = 16

(f - g)(2) = f(2) - g(2) = 8 - 8 = 0

(fg)(2) = f(2)g(2) = 8(8) = 64

I WILL GIVE BRAINLIEST!!!

consider the polynomial function q(x)=-2x^8+5x^6-3x^5+50

end behavior

Answers

Answer:

Use the degree and the leading coefficient to determine the behavior.

Falls to the left and falls to the right

Step-by-step explanation:

Solve the given system of differential equations by systematic elimination.
(D − 1)x+ (D² + 1)y = 1
(D² − 1)x+ (D + 1)y = 2

(x(t), y(t)) = (e^-t/2 [-5/3cos(√47/2)t - 125/3 sin(√47/2)t]+ 20/3 cos (3t) + 20/3 sin (3t)

Answers

Given system of differential equation is(D − 1)x+ (D² + 1)y = 1 ...

(i)(D² − 1)x+ (D + 1)y = 2 ...(ii)By using systematic elimination method, we have(D²+1)(D²−1)x+(D+1)(D−1)y=D²+1×1-(D+1)×1=0Simplifying the above equation, we get(D⁴-1)x=-(D-1)y...(iii)Applying D on both sides of (iii), we get D(D⁴-1)x=-(D-1 )DyD⁵x- Dx=-(Dy-y)or D⁵x+Dy=y ... (iv)Now applying D on (i), we get(D−1)Dx+(D²+1)Dy=0or D(D²+1)y=(1-D)x ...(v)Now applying D on (ii), we get(D²−1)Dx+(D+1)Dy=0or D(D+1)x=(1+D)y ...(vi)Now, substituting the value of x and y from equations (v) and (vi) in equation (iv), we getD⁵x+(1+D)Dx=(1-D)Dy D⁵x+(1+D)Dx=-(1-D)x ...(vii) Simplifying the above equation, we getD⁶x+2D⁴x+D²x+x=0or D²(D⁴+1)x+D²x=-x ...(viii)or D²(D⁴+2)x=-xor D⁴x+2x=-xor D⁴x=-3xNow using D on both sides, we get D⁵x=-3Dxor D⁶x=-3D²x

Now, substituting the value of D²x from equation (iii) in equation (i), we get(D-1)x+(D²+1)y=1 ...(i)⇒ (D-1)x+y=1 ...(ix)Now, substituting the value of D²x from equation (iii) in equation (ii), we get(D²-1)x+(D+1)y=2 ...(ii)⇒ -(D+1)x+y=0or (D+1)x-y=0 ...(x)From equation (ix) and (x), we have2x=1or x=1/2Now, substituting the value of x in equation (ix), we have D(1/2)+y=1or y=1-1/2=1/2Thus, the solution of the given system of differential equation is(x(t), y(t))=(e^(-t/2))[(-5/3)cos((sqrt(47)/2)t)-(125/3)sin((sqrt(47)/2)t)]+(20/3)cos(3t)+(20/3)sin(3t), (1/2)

Know more about differential equation:

https://brainly.com/question/32538700

#SPJ11

The change in water level of a lake is modeled by a polynomial function, W(x). Describe how to find the x-intercepts of W(x) and how to construct a rough graph of W(x) so that the Parks Department can predict when there will be no change in the water level. You may create a sample polynomial of degree 3 or higher to use in your explanations.
(Dividing and Solving Polynomials)

Answers

First. Finding the x-intercepts of  

Let  be the change in water level. So to find the x-intercepts of this function we can use The Rational Zero Test that states:

To find the zeros of the polynomial:

We use the Trial-and-Error Method which states that a factor of the constant term:

can be a zero of a polynomial (the x-intercepts).

So let's use an example: Suppose you have the following polynomial:

where the constant term is . The possible zeros are the factors of this term, that is:

.

Thus:

From the foregoing, we can affirm that  are zeros of the polynomial.

Second. Construction a rough graph of  

Given that this is a polynomial, then the function is continuous. To graph it we set the roots on the coordinate system. We take the interval:

and compute  where  is a real number between -2 and -1. If , the curve start rising, if not, the curve start falling. For instance:

Therefore the curve start falling and it goes up and down until  and from this point it rises without a bound as shown in the figure below

1. Study Activity 1 on p.301. Then complete the following using the Sampling Distribution
of a Sample Proportion web app.
i. Simulate taking a random sample of 100 voters from a large population of voters of
whom 54% voted for Brown, and record the number out of 100 that voted for Brown.
ii. Report the proportion of your sample that voted for Brown
iii. Insert below the Data Distribution generated by the web app.

Answers

According to the question the following using the Sampling Distribution are as follows :

1. Study Activity 1 on p.301. Then complete the following using the Sampling Distribution of a Sample Proportion web app.

i. Simulate taking a random sample of 100 voters from a large population of voters of whom 54\% voted for Brown, and record the number out of 100 that voted for Brown.

ii. Report the proportion of your sample that voted for Brown

iii. Insert below the Data Distribution generated by the web app.

i. Simulate taking a random sample of 100 voters from a large population of voters of whom 54\% voted for Brown, and record the number out of 100 that voted for Brown.

ii. Report the proportion of your sample that voted for Brown: [Insert the proportion value here]

iii. Insert below the Data Distribution generated by the web app:

[Insert the data distribution plot here]

Note: The actual values for the proportion and data distribution will vary based on the simulation results obtained from the Sampling Distribution of a Sample Proportion web app.

To know more about distribution visit-

brainly.com/question/30600552

#SPJ11

Arletta built a cardboard ramp
for her little brother’s toy cars.
Find the volume of this ramp.

Answers

Answer:

Volume = 525 in³

Step-by-step explanation:

Volume = 25 x 7 x 6 x 0.5 = 525 in³

Gerald and Wheatly, Applied Numerical Analysis ▶6. If e¹.3 is approximated by Lagrangian interpolation from the values for eº = 1, el = 2.7183, and e² = 7.3891, what are the minimum and maximum estimates for the error? Compare to the actual error.

Answers

Lagrangian interpolation is used to approximate the value of e¹.3 using three known values: eº = 1, el = 2.7183, and e² = 7.3891. We can find the minimum and maximum estimates for the error.

To approximate e¹.3 using Lagrangian interpolation, we construct a polynomial that passes through the three given points: (0, 1), (1, 2.7183), and (2, 7.3891). We can then evaluate this polynomial at x = 1.3 to estimate the value of e¹.3.

Using Lagrangian interpolation, the polynomial P(x) is given by:

P(x) = 1 * L₀(x) + 2.7183 * L₁(x) + 7.3891 * L₂(x),

where L₀(x), L₁(x), and L₂(x) are the Lagrange basis polynomials associated with the three data points.

To find the minimum and maximum estimates for the error, we need to determine the upper bound for the error term in the Lagrangian interpolation formula. The error term is given by:

E(x) = f(x) - P(x),

where f(x) is the actual function we are approximating (in this case, e^x).

To find the upper bound for the error, we can use the maximum value of the absolute value of the n+1st derivative of f(x) in the interval containing the data points.

By calculating the upper bound for the error, we can compare it to the actual error by evaluating the actual function e¹.3 and subtracting the approximation P(1.3) obtained from Lagrangian interpolation.

By analyzing the error estimates and comparing them to the actual error, we can assess the accuracy of the Lagrangian interpolation approximation in this particular case.

Learn more about Lagrangian here:

https://brainly.com/question/14309211

#SPJ11








Use the Laplace transform to solve the initial-value problem x" + 4 = f(t), x(0)=0, x'(0) = 0, if t < 5 f(t) = t25. 3 sin(t-5) if t > 5.

Answers

By applying the initial conditions and inverse Laplace transforming, we can obtain the solution x(t) = (1 - cos(2t))u(t-5) + (3 sin(t-5))u(t-5), where u(t) is the unit step function. Therefore, the solution to the initial-value problem is x(t) = (1 - cos(2t))u(t-5) + (3 sin(t-5))u(t-5)

Taking the Laplace transform of the given differential equation x" + 4 = f(t), we obtain the algebraic equation in the Laplace domain: s^2X(s) + 4sX(s) + 4 = F(s), where X(s) is the Laplace transform of x(t) and F(s) is the Laplace transform of f(t).

Next, applying the initial conditions x(0) = 0 and x'(0) = 0, we get X(0) = 0 and sX(0) = 0. Substituting these initial conditions into the Laplace domain equation, we have s^2X(s) + 4sX(s) + 4 = F(s), with X(0) = 0 and sX(0) = 0.

Now, let's consider the Laplace transform of f(t) using the given piecewise function. For [tex]t < 5, f(t) = t^2/5, and for t > 5, f(t) = 3sin(t-5).[/tex]Taking the Laplace transform of f(t) in each interval, we have [tex]F(s) = (1/s^3) + (3/s^2) for t < 5 and F(s) = (3/s^2) * (1/(s^2+1)) for t > 5.[/tex]

Substituting these Laplace transforms into the equation[tex]s^2X(s) + 4sX(s) +[/tex]4 = F(s), we can solve for X(s). Simplifying, we obtain [tex]X(s) = (1/s^3) + (3/s^2) / (s^2 + 4s + 4) + (3/s^2) * (1/(s^2+1)).[/tex]

To find the inverse Laplace transform of X(s), we can split it into partial fractions and apply the inverse Laplace transform formula. The solution is x(t) = (1 - cos(2t))u(t-5) + (3 sin(t-5))u(t-5), where u(t) is the unit step function.

Therefore, the solution to the initial-value problem is x(t) = (1 - cos(2t))u(t-5) + (3 sin(t-5))u(t-5), where u(t) is the unit step function that ensures the piecewise function is activated at t = 5.

Learn more about Laplace transform here:

https://brainly.com/question/31040475

#SPJ11

The Taylor polynomial P, = (-10 * 9 about x = 0 is used to approximate the value of the function f at x=1 Find the value that verifies 5p (1)-(1)-500 n=1 n! Pa 1 384 1 384 0 ਤਕ OP PA 1 6144 PA 6144

Answers

The value that verifies 5p (1)-(1)-500 is -124.04

To approximate the value of the function f at x = 1 using the Taylor polynomial Pₙ = (-10)^n/ n! about x = 0, we need to find the value of P₅(1).

First, let's compute the derivatives of f(x) = e^x up to the fifth derivative:

f'(x) = e^x

f''(x) = e^x

f'''(x) = e^x

f''''(x) = e^x

f⁽⁵⁾(x) = e^x

Now, let's evaluate these derivatives at x = 0:

f(0) = e^0 = 1

f'(0) = e^0 = 1

f''(0) = e^0 = 1

f'''(0) = e^0 = 1

f''''(0) = e^0 = 1

f⁽⁵⁾(0) = e^0 = 1

Using these values, we can compute the Taylor polynomial P₅(x):

P₅(x) = f(0) + f'(0)(x - 0) + f''(0)(x - 0)²/2! + f'''(0)(x - 0)³/3! + f''''(0)(x - 0)⁴/4! + f⁽⁵⁾(0)(x - 0)⁵/5!

P₅(x) = 1 + 1x + 1x²/2! + 1x³/3! + 1x⁴/4! + 1x⁵/5!

Now, let's evaluate P₅(1):

P₅(1) = 1 + 1(1) + 1(1)²/2! + 1(1)³/3! + 1(1)⁴/4! + 1(1)⁵/5!

P₅(1) = 1 + 1 + 1/2 + 1/6 + 1/24 + 1/120

P₅(1) = 227/120

Therefore, the value that verifies 5P₅(1) - (1) - 500 is:

5P₅(1) - (1) - 500 = 5 * (227/120) - 1 - 500

= 1135/120 - 1 - 500

= 1135/120 - 120/120 - 60000/120

= (1135 - 120 - 60000)/120

= -59485/120

= -124.04

So, the value that verifies 5P₅(1) - (1) - 500 is approximately -124.04.

To learn more about Taylor polynomial

https://brainly.com/question/27137596

#SPJ11

One angle measures 19° and another angle measures (4d − 9)°. If the angles are complementary, what is the value of d?

d = 7
d = 20
d = 25
d = 42.5

Answers

Answer:

d = 20

Step-by-step explanation:

90-19 = 71

(4d-9) = 71

4d = 80

d = 20

Answer:

d = 20

Step-by-step explanation:

Complementary angles are two angles that add up to 90°.

We know that one angle is 19° and the other is (4d − 9)°. So, we can set up the equation:

19 + (4d − 9) = 90.

Solving for d, we get:

19 + (4d − 9) = 90

19 + 4d − 9 = 90

4d + 10 = 90

4d = 80

d = 20

Therefore, the value of d is 20.

1. Find the equation of the parabola satisfying the given conditions.

Focus: (3,6); Directrix: x=−1

A. (x−1)2=8(y−6)

B. (y−6)2=8(x−1)

C. (x−1)2=−8(y−6)

D. (y−6)2=−8(x−1)


2. Find the equation of the parabola satisfying the given conditions.

Focus: (−6,3); Directrix: y=1

A. (y−2)2=4(x+6)

B. (x+6)2=4(y−2)

C. (x+6)2=−4(y−2)

D. (y−2)2=−4(x+6)


3. Find the equation of an ellipse that has foci at (−1,0) and (4,0), where the sum of the distances between each point on the ellipse and the two foci is 9.

A. (x+1)2+y2−−−−−−−−−−−√+(x−4)2+y2−−−−−−−−−−−√=9

B. (x−1)2+y2−−−−−−−−−−−√+(x+4)2+y2−−−−−−−−−−−√=9

C. (x+1)2+y2−−−−−−−−−−−√+(x−4)2+y2−−−−−−−−−−−√=81

D. (x−1)2+y2−−−−−−−−−−−√+(x+4)2+y2−−−−−−−−−−−√=81


4. Find the equation of a hyperbola that has foci at (−1,0) and (4,0), where the difference of the distances between each point on the ellipse and the two foci is 5.

A. (x+1)2+y2−−−−−−−−−−−√−(x−4)2+y2−−−−−−−−−−−√=25

B. (x−1)2+y2−−−−−−−−−−−√−(x+4)2+y2−−−−−−−−−−−√=5

C. (x−1)2+y2−−−−−−−−−−−√−(x+4)2+y2−−−−−−−−−−−√=25

D. (x+1)2+y2−−−−−−−−−−−√−(x−4)2+y2−−−−−−−−−−−√=5

Answers

Answer:

CABD

Step-by-step explanation:

With which information can you construct more than one triangle?
A the measurements of two angles
B the measurements of two angles and the length of the included side
C the measurements of all the angles
D the lengths of two sides and the measurement of the included angle

Answers

B because side length and angle measure is given

Answer:

B and D

Explanation:

hurry !!!! I need help​

Answers

Answer:

x = 66

Step-by-step explanation:

m<5 + m<6 = 180

2x + 48 = 180

2x = 132

x = 66

HELPPPPPPPPPPPPPPP!!!!!!!

Answers

i think it’s -2 and 10 but sorry if i am wrong btw the 10 is on the left and -2 on the right

Answer:

I feel like the answers are 10 and -2

Step-by-step explanation:

Hope this helps! Have a great day and good luck! :)

Ariana orders 4 large pizzas and 1 order of breadsticks The total for her order is $34.46. Emily orders 2 large pizzas and 1 order of breadsticks. $18.48 is the total for her order. Determine the order for 1 large pizza and 1 order of breadsticks?

Answers

4p + 1b = 34.46
2p + 1b = 18.48

Solve either equation for b

2p + b = 18.48
b = 18.48 - 2p

Substitute into first equation
4p + 18.48 - 2p = 34.46
2p + 18.48 = 34.46
2p = 15.98
p = $7.99

Plug back into either equation to find cost of breadsticks

2(7.99) + b = 18.48
15.98 + b = 18.48
b = $2.50

One pizza + breadstick
$7.99 + $2.50
$10.49 :)

Can someone plz help me with the answer

Answers

Step-by-step explanation:

[tex] \frac{12 \times 8}{2} + {( \frac{8}{2}) }^{2} \pi = \\ = 48 + 16\pi[/tex]

98.24

What is the Quartile 1 for the Box & Whisker Plot below?


PLSS HELP

Answers

Answer:

17

Step-by-step explanation:

The lower quartile Q₁ is positioned at the left side of the box.

The value at the left is Q₁ is 17

HELP HELP PLEASE WRING ANSWERS OR LINKS WILL BE REPORTED ​

Answers

Answer:

$0.24 per orange

Step-by-step explanation:

$3.84 / 16 = 0.24

suppose 40% of adults in the u.s. say they get their financial advice from family members. a random sample of 8 adults is selected. what is the probability at least 5 of the 8 say they get their financial advice from family members?

Answers

The probability that at least 5 of the 8 adults say they get their financial advice from family members can be calculated using binomial probability distribution.

Formula for binomial probability distribution is:P(X=k) = nCk * pk * (1-p)n-kwhere, P(X=k) is the probability of k successes in n independent trials, p is the probability of success in one trial, q=1-p is the probability of failure in one trial, nCk is the combination of k successes in n independent trials.

In this case, the probability of success is p=0.4 as 40% of adults in the US say they get their financial advice from family members.

Therefore, the probability of failure is q=1-0.4=0.6.So, P(X ≥ 5) = P(X = 5) + P(X = 6) + P(X = 7) + P(X = 8)P(X=k) = nCk * pk * (1-p)n-kwhere, n=8, p=0.4, q=0.6For k = 5, nCk = 8C5P(X = 5) = 8C5 * (0.4)5 * (0.6)3= 0.2787

For k = 6, nCk = 8C6P(X = 6) = 8C6 * (0.4)6 * (0.6)2= 0.1960For k = 7, nCk = 8C7P(X = 7) = 8C7 * (0.4)7 * (0.6)1= 0.0575For k = 8, nCk = 8C8P(X = 8) = 8C8 * (0.4)8 * (0.6)0= 0.0030Therefore,P(X ≥ 5) = P(X = 5) + P(X = 6) + P(X = 7) + P(X = 8) = 0.2787 + 0.1960 + 0.0575 + 0.0030 = 0.5352

Hence, the probability that at least 5 of the 8 adults say they get their financial advice from family members is 0.5352.

To know about financial, visit:

https://brainly.com/question/989344

#SPJ11

Given information that 40% of adults in the US say they get their financial advice from family members. We have to find the probability of at least 5 of the 8 say they get their financial advice from family members.

Hence, the required probability is 0.7530.

The probability of getting financial advice from family members is 40%. Let X be the number of people out of 8, who get their financial advice from family members. Here, X follows a binomial distribution with parameters n = 8 and p = 0.4. The probability of getting atleast 5 people getting their financial advice from family members is

P(X ≥ 5) = P(X = 5) + P(X = 6) + P(X = 7) + P(X = 8)

Using binomial distribution formula, we get

P(X = x) = ${n\choose x}p^xq^{n-x}$

Where, n = 8, p = 0.4, q = 0.6

The probability of getting exactly 5 people out of 8 getting their financial advice from family members

P(X = 5) = ${8\choose 5} 0.4^5 (0.6)^{8-5}$

= 0.27869

The probability of getting exactly 6 people out of 8 getting their financial advice from family members

P(X = 6) = ${8\choose 6} 0.4^6 (0.6)^{8-6}$

= 0.29360

The probability of getting exactly 7 people out of 8 getting their financial advice from family members

P(X = 7) = ${8\choose 7} 0.4^7 (0.6)^{8-7}$

= 0.16493

The probability of getting exactly 8 people out of 8 getting their financial advice from family members

P(X = 8) = ${8\choose 8} 0.4^8 (0.6)^{8-8}$

= 0.01678

Therefore, the probability of getting at least 5 people getting their financial advice from family members is

P(X ≥ 5) = 0.27869 + 0.29360 + 0.16493 + 0.01678

= 0.7530

Hence, the required probability is 0.7530.

To know more about probability visit

https://brainly.com/question/32004014

#SPJ11

A cube with an edge of length s has a volume of 27 units.
What is the length of s?

Answers

Answer:

s = 3

Step-by-step explanation:

The volume formula for a cube is V = s^3, where “s” is the edge length. Since we know the volume and need to find “s,” we just do the inverse operation for an exponent, which is a radical. The cubed root of 27 is 3, so there’s your answer! Hope this is helpful & accurate. Best wishes.

A chocolate muffin recipe serves 12 people. An oatmeal raisin cookie recipe serves 36 people. A lemon cake recipe serves 16 people. Each recipe needs 2 eggs. The muffins need 3 cups of flour, while the cookies need 2 cups. The cake uses 2 lemons and the cookies use 1 cup of raisins and oatmeal. Each recipe needs 1 cup of sugar and milk. If we need to make cookies for 48 people, how much flour is needed?

Answers

To make cookies for 48 people, the recipe requires 6 cups of flour.

The oatmeal raisin cookie recipe serves 36 people and requires 2 eggs and 2 cups of flour. Since we need to make cookies for 48 people, we can calculate the amount of flour required as follows:

36 people → 2 cups of flour

1 person → (2 cups of flour) / (36 people) = (1/18) cups of flour

To make cookies for 48 people:

48 people × (1/18) cups of flour = 2.67 cups of flour

Therefore, to make cookies for 48 people, we need approximately 2.67 cups of flour.

Note: Since the result is a fraction of a cup, it is advisable to round up to the nearest whole number, so in this case, 3 cups of flour would be needed.

To learn more about “fractions” refer to the https://brainly.com/question/78672

#SPJ11

The students in a club are selling flowerpots to raise money.Each flowerpot sells for $15.

Part A

Write an expression that represents The total amount of money, in dollars, The students raise from selling flowerpots.
Answer your expression in the box provided. Enter only your expression.Please hurry!

Answers

Answer:

y = 15x

Step-by-step explanation:

For every flower pot purchased (y), the quantitity of the price (x) will go up by $15.

Answer:

y = 15x

Step-by-step explanation:

Please answer correctly! I will mark you Brainliest!

Answers

Answer:

V=385 cubic units

Step-by-step explanation:

The volume of a rectangular prism is given by the formula [tex]V=lwh[/tex], where l is the length, w is the width, and h is the height. Our dimensions are 5, 7, and 11. So, we have to multiply them to find the volume.

5 × 7 × 11 = 385

Thus, the volume of the rectangular prism is 385 cubic units.

Solve the following differential equation by using Laplace transform method. y" +2y' +y = cos2t where y(0)=1 y'(O)=1.

Answers

The solution to the given differential equation with the initial conditions y(0) = 1 and y'(0) = 1 is:

[tex]y(t) = -e^{-t}/10 + (11/10)t*e^{-t} + (1/10)cos(2t) + (3/10)sin(2t)[/tex]

To solve the given differential equation using Laplace transform, we will apply the Laplace transform to both sides of the equation and then solve for the transformed variable.

Let's denote the Laplace transform of y(t) as Y(s).

Taking the Laplace transform of both sides of the differential equation, we get:

[tex]s^2Y(s) + 2sY(s) + Y(s) = (s^2 + 2s + 1)/(s^2 + 4)[/tex]

Now, let's solve for Y(s):

[tex]Y(s)(s^2 + 2s + 1) = (s^2 + 2s + 1)/(s^2 + 4)\\Y(s) = (s^2 + 2s + 1)/(s^2 + 4)(s^2 + 2s + 1)[/tex]

Factoring the denominator:

[tex]Y(s) = (s^2 + 2s + 1)/((s + 1)^2(s^2 + 4))[/tex]

Now, we need to decompose the fraction into partial fractions. Let's express the numerator in terms of A, B, C, and D:

[tex]s^2 + 2s + 1 = A/(s + 1) + B/(s + 1)^2 + (Cs + D)/(s^2 + 4)[/tex]

To find the values of A, B, C, and D, we can equate the numerators:

[tex]s^2 + 2s + 1 = A(s + 1)(s^2 + 4) + B(s^2 + 4) + (Cs + D)(s + 1)^2[/tex]

Expanding and equating coefficients:

[tex]s^2 + 2s + 1 = A(s^3 + 5s^2 + 4s) + B(s^2 + 4) + (C(s^2 + 2s + 1) + D(s + 1)^2)[/tex]

Simplifying:

[tex]s^2 + 2s + 1 = (A + C)s^3 + (5A + C + D)s^2 + (4A + 2C + D)s + (4A + D)[/tex]

Equating coefficients:

A + C = 0 (coefficient of [tex]s^3[/tex])

5A + C + D = 1 (coefficient of [tex]s^2)[/tex]

4A + 2C + D = 2 (coefficient of s)

4A + D = 1 (constant term)

Solving these equations simultaneously, we find A = -1/10, B = 11/10, C = 1/10, and D = 3/10.

Now, substituting these values back into Y(s):

[tex]Y(s) = (-1/10)/(s + 1) + (11/10)/(s + 1)^2 + (1/10)(s + 3)/(s^2 + 4) + (3/10)/(s^2 + 4)[/tex]

To find y(t), we need to take the inverse Laplace transform of Y(s). Fortunately, we can use a Laplace transform table to find the inverse Laplace transform of each term.

The inverse Laplace transform of (-1/10)/(s + 1) is [tex]-e^{-t}/10.[/tex]

The inverse Laplace transform of (11/10)/(s + 1)² is (11/10)t*[tex]e^{-t}.[/tex]

The inverse Laplace transform of (1/10)(s + 3)/(s² + 4) is (1/10)cos(2t).

The inverse Laplace transform of (3/10)/(s² + 4) is (3/10)sin(2t).

Combining these results, the solution y(t) is:

[tex]y(t) = -e^{-t}/10 + (11/10)t*e^{-t} + (1/10)cos(2t) + (3/10)sin(2t)[/tex]

Therefore, the solution to the given differential equation with the initial conditions y(0) = 1 and y'(0) = 1 is:

[tex]y(t) = -e^{-t}/10 + (11/10)t*e^{-t} + (1/10)cos(2t) + (3/10)sin(2t)[/tex]

To learn more about Laplace transform visit:

brainly.com/question/14487937

#SPJ11

Other Questions
Sam's is considering a Project with the following cash flows:Initial Investment=$1,748.80Cash flow from years 1 to 6 = $500 per yearDetermine the internal rate of return "IRR" of the project:Answer: IRR=____________________________________Show Formula And Procedure (Explanation) As the training and development coordinator for the Milton Coffee House and Restaurant, you are tasked with creating a presentation of 8 slides for an upcoming kitchen staff meeting. The training will focus on the importance of time management skills by incorporating some of the strategies listed above. PLEASE ANSWER ASAP FOR BRAINLEST!!!!!!!!!!!!!!!!!!!!!!!!!!!!!!!!!! LINKS WILL BE REPORTED!!Similarities in DNA sequence can be seen in all speciesA. Anatomical EvidenceB. Homologous StructureC. Analogous StructuresD. Vestigial StructuresE. FossilF. EmbryologyG. DNA/Molecular Evidence It is commonly recognized that a poor environment (for example, poverty, low educational level of parents) and the presence of stressors (for example divorce of parents, abuse) are associated with lower IQs in children. Recent research indicates that a child's IQ is related to the number of such risk factors in the children's background, regardless of which specific factors are present. Below are the numbers of risk factors and the IQs of 12 children.ChildRisk factors (x)IQ (y)A3112B682C0105D5102E1115F1101G594H489I398J3109K491L2107A. Draw a scatterplot of the data. What does the plot tell you? Determine the regression equation. Be sure to include an interpretation of each parameter in the regression equation.B. What IQ do you predict for James, who has 5 risk factors, versus Elizabeth, who has only 2? Consider the first three terms of the sequence below.14000, 12600, 11340Complete a recursively-defined function to describe this sequence.f(1) = _____f(n) = f(n - 1) ___, for n 2The next term in the sequence is ____. Which of these are examples of INFINITIVE verb? (Select ALL that apply) GEOMETRY find the angle of PRQ NO LINKS!! 7. The area of a triangle is 2.1 square feet. Its base is 2 feet. What is its height Can you give me the answer to this A filing cabinet is 2 yards long by 2 feet wide by 48 inches high. What is the volume of the filing cabinet in cubic inches? Enter your answer in the box. ASAP PLEASEType in your response.What is the area of kite ABCD? sq yd what are three transitional word phrases? (RL5) Which best describes the structure of this passage? *O A. lively dialogueO B. descriptive narrationO C. painful flashbackO D. shocking foreshadowinghelp !!!! a)Estimate a model relating annual salary to firm sales and market value. Make themodel of constant elasticity variety for both independent variables. Write the resultsout in equation form (s.e. under parameter estimates). summary (lm(formula= salary sales + mktval, data = ceosal2)) Call: lm(formula = salary sales + mktval, data = ceosal2) Residuals: Coefficients: segnitr. coues: v Residual standard error: 535.9 on 174 degrees of freedom Multiple R-squared: 0.1777, Adjusted R-squared: 0.1682 F-statistic: 18.8 on 2 and 174 DF, p-value: 4.065e08 > lm(formula = lsalary lsales + lmktval, data = ceosal2) Call: lm(formula = lsalary lsales + lmktval, data = ceosal2) Coefficients: (Intercept) 4.6209Lsales 0.1621Lmktval 0.1067b) A friend of yours is about to start as a CEO at a firm. She is thinking of asking for$500.000 as annual salaries. The firm sales last year was $5.000.000 and the marketvalue of the firm is $20 million. According to your model from part (a) would she beasking too much? What are the expected salaries according to the model? 3. The third side of a triangle measures (3x-5) cm. If the length of the midline is 14 m,what is ?A 8C. 10B. 9D. 11 For those counselors who work with addictive and comorbid clients within a Twelve Step model, they should be familiar with which of the following texts?1. Feeling Good Handbook2. Sponsors Guide3. Big Book4. Hazelton Dictionary What actions has amazon taken to fight walmart on walmart's turf? how has walmart retaliated? comment on the risks of these actions by both walmart and amazon. If an equipment is damaged during unpacking or installation, the repair expenses are added to its cost O True O False ________ is the application of standards of moral behavior tobusiness situations.Multiple ChoiceBusiness structuralismBusiness contingenceBusiness ethicsBusiness sourcing T/F. the force between the two wires 1. pulls the wires together.